6. How many 1/2 hour periods will make up 4 hours?

Answers

Answer 1

Answer: it will take 8 periods of 1/2 hour to make up 4 hours

Step-by-step explanation:

Answer 2

It will take 8 periods of 1/2 hour to make up 4 hours.

What are Arithmetic operations?

Arithmetic operations can also be specified by the subtract, divide, and multiply built-in functions.

Half an hour is 30 minutes,

There are 60 minutes in an hour.

4 hours=240 minutes.

240/30 = 8

Hence, 8 periods of 1/2 hour to make up 4 hours.

Learn more about Arithmetic operations

https://brainly.com/question/25834626

#SPJ5


Related Questions

Identify the property being illustrated.
6x + 7 = 7 + 6x

Answers

Answer:

Commutative property of addition

Which table of ordered pairs represents a proportional relationship? ​

Answers

Answer:

2nd one

For example 5 squared is 25, 7 Squared is 49, and 9 Squared is 81

Find the midpoint of the line segment with the given endpoints (-3,1 -2,8) (-4.92,-3.3)

Answers

Answer:

Step-by-step explanation:

(x₁ , y₁) = (-3.1  , -2.8)   & (x₂ , y₂) = (-4.92 , - 3.3)

[tex]Midpoint=(\frac{x_{1}+x_{2}}{2},\frac{y_{1}+y_{2}}{2})\\\\\\=(\frac{-3.1-4.92}{2},\frac{-2.8-3.3}{2})\\\\\\=(\frac{-8.02}{2},\frac{-6.1}{2})\\\\\\=( -4.01 , -3.05)[/tex]

PLEASE HELP IF YOU KNOW ​

Answers

Your answer is going to be A

Step-by-step explanation:

I need help and it’s due today

Answers

Answer:

5.63

Step-by-step explanation:

Angle NLM is equal to KLM - KLN.

Since KLM = 137° and KLN = 47°, NLM = °90. (Maybe it doesn't look like a right angle, but the math doesn't lie.)

Since NLM = 16y, then 90 = 16y.

Dividing both sides by 16, we find that:

y = 5.625

Answer:

y = 5.63

Explanation:

This answer is 5.23 because ∠KLN plus ∠NLM are equal to ∠KLM or 137°.

If you subtract 47 from 137 you will get 90. If you divide 90 by 16 you will get 5.625 which is rounded to 5.63.

Hi, can someone answer this?

Answers

Answer:

I think its D

Step-by-step explanation:

the 2 on top of the b equals 400.

400 times 400 = 160,000

( Hope this helps! )

evaluate 7m+2n-8p/n m=4, n=2, and p=1.5

Answers

Steps to solve:

7m + 2n - 8p/n

~Substitute

7(4) + 2(2) - 8(1.5)/2

~Simplify

28 + 4 - 12/2

~Simplify

28 + 4 - 6

~Add

32

~Subtract

26

Best of Luck!

The sum of six number is 33. What is their mean?

Answers

Answer:

mean = 33/6 =5.5

Step-by-step explanation:

Mean = sum of numbers / the amount of numbers(like how many numbers are there)

Mean= 33/6

=5.5

So mean is 5.5

If the temperature in Texas dropped 35 degrees in 7 hours on Tuesday. What is the change over time?

Answers

Answer:

5 degrees every hour.

Step-by-step explanation:

35/7=5

Answer: 5

Step-by-step explanation:

35 degrees / 7 hours = 5 degrees per hour

Find the distance of segment AB

A(-7,6) B(4,-2)

Answers

Answer:

13.6

Step-by-step explanation:

WILL AWARD BRAINLIEST. How many gallons of water fill a tank that is 16.5 ft long, 7.5 ft wide, and 8 ft high? Assume 1 ft3 is 7.5 gallons of water.

Answers

Answer:

7,425 gallons of water

Step-by-step explanation:

Given:

A tank of,

Length = 16.5 ft

Width = 7.5 ft

Height = 8 ft

1 ft³ = 7.5 gallons of water

Required:

Gallons of water that will fill the tank

SOLUTION:

Find the volume of the tank:

Volume of the tank = length*width*height

Volume = 16.5*7.5*8 = 990 ft³

Convert 990 ft³ to gallons of water:

1 ft³ = 7.5 gallons

990 ft³ = [tex] 990*7.5 [/tex]

= 7,425 gallons of water

7,425 gallons of water will fill the tank of the given measurement.

Amount of water in tank is 7,425 gallon

Volume base question:

Given that;

Sides of tank = 16.5 ft, 7.5 ft, 8 ft

1 ft³ = 7.5 gallon

Find:

Amount of water in tank

Computation:

Volume of tank = [(16.5)(7.5)(8)]

Volume of tank = 990 ft³

Amount of water in tank = [Volume of tank][7.5]

Amount of water in tank = [990][7.5]

Amount of water in tank = 7,425 gallon

Find out more information about 'Volume'.

https://brainly.com/question/1578538?referrer=searchResults

Suppose a necklace is made from 18-karat gold and weighs 54 grams. Find the weight, in grams, of the pure gold in the necklace.

Answers

I work part time at a jeweler, that necklace would be worth about 3.3K and then also you didn't provide what the effect is. Wouldn't the necklace remain as 54 grams?

i’m stuck on this question

Answers

Answer: Dividing the number by 1000

When you move the decimal, add a 0.
The decimal is moved 3 times, there are 3 zeros. When you move left, the number is getting smaller so it is division.

Hope this helps ʕ•ᴥ•ʔ

Answer:

See below

Step-by-step explanation:

So you are moving the decimal to the right, right?

This action would make the number bigger.

For this explanation I will use the example 7.2

So lets move the decimal 3 to the right.

You get 7200.

Now try dividing 7200 by 100 = 72

This did not get us the original # 7.2

Now lets divide 7200 by 1000 = 7.2

Now that we got the original #, switch the operation and multiply 7.2 by 1000

7.2 x 1000 = 7200

Answer = multiplying the number by 1,000

HELP I HAVE TO SUBMIT IN 1 HOUR

Answers

Answer:

Step-by-step explanation:

Volume = Base * height

V=2(7 1/3)

V=44/3 OR 14 2/3  Or 14.666

12x + 9 - 3x - 4. Please go fast!!!!

Answers

Answer:

9x +5

Step-by-step explanation:

combine like terms!

9x +5 combine like terms then subtract

There are 15 different pieces of cheese on the plate. Is it possible by dividing only one piece to divide all into two groups with equal weight and equal amount of pieces in each?

Answers

Answer:

It is possible.

Step-by-step explanation:

Each piece of cheese can be divided into two equal pieces.

Therefore, the 15 different pieces can produce 30 (15 * 2) equal pieces.

Then there will be 15 pieces in each group.

Provided during the division, it is ensured that equal weights were allocated to each piece, then each group will have the same weight as the other group.

Note that the division of the 15 pieces of cheese into two equal parts looks like the only way to have equal weight of two groups of the delicacy.

Simplify to create an equivalent expression.
4(-15 – 3p) - 4(-p+5)
Choose 1 answer:
(Α)
-8p - 80
B
- 13p - 80
-8p + 80
8p - 80

Answers

Answer:

A) I got this question right on my assignment

the answer for this question is A

As a child gets older, will they be able to solve a puzzle faster? What is the dependent variable?

Answers

Answer:

the dependant variable is the time

A dog walking along some train tracks meets a train every 4 minutes and the train catches up the dog every 12 minutes. Find the value of x if the trains leave the endpoint of the route every x minutes.

Answers

Answer and Step-by-step explanation:

Dog meets the train in = 4 minutes

Train catches up the dog in = 12 minutes

Let x be the time when train leave the route,

Train catch up the dog in 12 minutes, the endpoint of route is x and the dog meet up the train in 4min, so the equation becomes:

                       12 - x = 4

               To find out the value of x, subtract 12 from both sides, we get

                        x = 4 – 12

                         -x = -8

Which gives x = 8, when train leaves the endpoint of route.

AVENGERS: The Avender movie made $1.6 million in ticket sales. Its sequel made $0.8 million in ticket sales. How much more than the first movie make than the sequel. (a) Write and equation and solve it (b) Write your answer in a complete sentence in context of the problem

Answers

Answer:

0.8 million

Step-by-step explanation:

0.8 million because the first movie made 1.8 mill iou on and the question is asking how much more the first one made than the second one.

Jeremy spends $4.00 for breakfast and then spends $4.00 for lunch

Answers

Answer:

He spends 8 dollars

Step-by-step explanation:

4 + 4 = 8

Hopefully this helps you :)

pls mark brainlest ;)

Help quickly please hurry i am almost done

Answers

Answer:

Might be wrong but i got c( NOPE NOT C ITS A

Step-by-step explanation:

You are the CEO of a big company. You are paid an annual salary, plus a bonus of the company's profits over $5,000,000,000. Consider the two functions: g(x) = x - 5,000,000,000 ; f(x) = 0.01x which of the following represents your bonus?
a. (g ⚬ f)(x)
b. (f ⚬ g)(x)

Answers

Answer:

(g ⚬ f)(x)

Step-by-step explanation:

(g ⚬ f)(x)

1-89. On graph paper, draw the quadrilateral with vertices A(–1, 3), B(4, 3), C(–1, –2), and D(4, –2).

Answers

Step-by-step explanation:

this is the answer

in graph

please mark me as the brainliest I really need it

Find the number that should be added to the expression to make it a perfect square. k^2-5k

Answers

(K-5/2)^2-25/4 is your answer

Answer:

25/4

Step-by-step explanation:

if the population there increased by 15,183 people every week how many more people would be in Mexico city everyday?

Answers

Answer:

The answer is 2169

Step-by-step explanation:

You would do 15,183 x 7 because there are seven days in a week.

If I have a 9ft plank how many 3/4 pieces can I get

Answers

you can get 12 pieces I think

What is another nonterminating decimal number that can not be written as a fraction?

Answers

Answer:

  0.6066066606666066666...(increasing numbers of 6)

Step-by-step explanation:

One way to make a decimal non-repeating is to extend a pattern by one digit each time you write the pattern. That is the structure of the number Sophia and Nora are looking at. It is also the structure of the number in your question ...

  https://brainly.com/question/17546271

__

Any irrational number will have a non-repeating decimal: √2, for example.

Warren had d dollars. He bought 5 ice creams which cost x dollars each. How much money does he have left?

Answers

Answer:

The answer is: d-5x

Step-by-step explanation:

What I like to do is rewrite the problem with numbers instead of variables.

Example: Warren had 20 dollars. He bought 5 ice creams which cost 2 dollars each. How much money does he have left?

He has 20 dollars, 5x2=10 he spent 10$ on the ice cream so you have to subtract how much he had in the beginning to how much he spent, the answer with numbers would be: 20-5x2

With variables it would be d-5x

regi received a 3.5 raise which raised his yearly income by $910. what is his new income?

Answers

Answer:

His new income is $26,910.

Step-by-step explanation:

As Regi has received an increase in his annual salary that has represented $ 910 in his payment, to determine his new salary the following calculation must be performed:

910 / 3.5 = 1% salary

260 = 1% salary

260 x 100 = previous salary

26,000 = previous salary

26,000 + 910 = current salary

26,910 = current salary

Therefore, his new income is $26,910.

Other Questions
Which situation can be represented by the inequality 3.35x + 25 < 6.75x + 5? -16 = -5x + x please help I need help[tex] - 3n + 7 + 2n - 8[/tex]this is due today a charge of 12 passes through the filament of a car head lamp bulb in 4 s. what is current? Your class is collecting macroinvertebrates (insects & other small animals found in water) to determine the pollution level in a local creek. Each macroinvertebrate that you collect has a 'Pollution Rating' between 0 and 10. A rating of 0 means the animal is usually found in very polluted water. A rating of 10 means it is only found in very clean water. A total of 100 insects are caught, and the total number of points is added up. Your data supports all BUT one of the following conclusions. P Mar1 The table shows pairs of figures that are related to each other in the same way.Figure 1 Figure 2OoWhich conjecture best describes how the second figure in each set is related to the first?A) Figure 2 is always a horizontal reflection of Figure 1.B) Figure 2 is always a vertical reflection of Figure 1.C) Figure 2 is always a 90 clockwise rotation of Figure 1.D) Figure 2 is always a 180 rotation of Figure 1. Snow is falling steadily in syyracuse ny. After two hours 4inches of snow has fallen. write an equation that gives the amount of snow that has fallen after x hours at this rate Is government important in economics? Read the paragraph.My brother Steven has wanted to drive ever since he was old enough to know what a car is. Hehas no trouble steering, but his feet can't apply enough pressure to the gas pedal or the brake.Do you know of a mechanism he could use to make a car completely hand controlled?The root mechan means "machine." The suffix -ism means "act, practice, or process." Basedon this information, what does mechanism mean in the paragraph? A class has 50 students. 20 students are playing basketball and rest are playing volleyball. The percent of the class playing basketball is? a) 20 percent b) 40 percent c) 60 percent 1112131415161718TIME REMA02:29:How did industrialization contribute to the changing role of women in the late 1800s?O Women who worked outside the home were allowed to vote in state and national elections.O New appliances helped women complete household chores faster so they could pursue other interests.O New appliances signaled the end of housework, freeing women to work outside the home.O Women who worked outside the home were able to hold public office. Evaluate the expression for x = -3 and y = 43x2 2xy ok can someone tell me if this is a good thesis statement. "Video games have many great effects on our everyday lives, although the negative effects have people questioning them" i feel like its too bland Why was cattle driving profitable? If all other things remain equal, which of the following changes to a closed circuitwould result in increased resistance to the flow of electric current?Reducing the temperature of the conductorReducing the thickness of the conductorReplacing the conductor with a more conductive materialReducing the length of the conductor A class has 30 boys and 20 girls. The average age ofboys is 11 yrs and girls 12 What is theaverage age of the whole class." could you help me convert 10 miles to feet? 5x + 3y = 25 & x - 3y = 15. (USE THE GRAPH) Vectors B and C and are shown in the graph. Vector B is multiplied by the scalar 92. What angle does the new vector make with the x-axis? i cant figure this out